2021 JMPSC Accuracy Problems/Problem 11

Revision as of 21:30, 10 July 2021 by Samrocksnature (talk | contribs) (Created page with "==Problem== If <math>a : b : c : d=1 : 2 : 3 : 4</math> and <math>a</math>, <math>b</math>, <math>c</math>, and <math>d</math> are divisors of <math>252</math>, what is the ma...")
(diff) ← Older revision | Latest revision (diff) | Newer revision → (diff)

Problem

If $a : b : c : d=1 : 2 : 3 : 4$ and $a$, $b$, $c$, and $d$ are divisors of $252$, what is the maximum value of $a$?

Solution

asdf